Difference between revisions of "2023 AMC 12A Problems/Problem 16"

(Created page with "Hey the solutions will be posted after the contest, most likely around a couple weeks afterwords. We are not going to leak the questions to you, best of luck and I hope you ge...")
 
(import)
Line 1: Line 1:
Hey the solutions will be posted after the contest, most likely around a couple weeks afterwords. We are not going to leak the questions to you, best of luck and I hope you get a good score.  
+
==Problem==
 +
Consider the set of complex numbers <math>z</math> satisfying <math>|1+z+z^{2}|=4</math>. The maximum value of the imaginary part of <math>z</math> can be written in the form <math>\tfrac{\sqrt{m}}{n}</math>, where <math>m</math> and <math>n</math> are relatively prime positive integers. What is <math>m+n</math>?
  
-Jonathan Yu
+
<math>\textbf{(A)}~20\qquad\textbf{(B)}~21\qquad\textbf{(C)}~22\qquad\textbf{(D)}~23\qquad\textbf{(E)}~24</math>
 +
 
 +
==Solution==
 +
 
 +
 
 +
==See Also==
 +
{{AMC12 box|year=2023|ab=A|num-b=16|num-a=17}}
 +
{{MAA Notice}}

Revision as of 23:51, 9 November 2023

Problem

Consider the set of complex numbers $z$ satisfying $|1+z+z^{2}|=4$. The maximum value of the imaginary part of $z$ can be written in the form $\tfrac{\sqrt{m}}{n}$, where $m$ and $n$ are relatively prime positive integers. What is $m+n$?

$\textbf{(A)}~20\qquad\textbf{(B)}~21\qquad\textbf{(C)}~22\qquad\textbf{(D)}~23\qquad\textbf{(E)}~24$

Solution

See Also

2023 AMC 12A (ProblemsAnswer KeyResources)
Preceded by
Problem 16
Followed by
Problem 17
1 2 3 4 5 6 7 8 9 10 11 12 13 14 15 16 17 18 19 20 21 22 23 24 25
All AMC 12 Problems and Solutions

The problems on this page are copyrighted by the Mathematical Association of America's American Mathematics Competitions. AMC logo.png